LSAT and Law School Admissions Forum

Get expert LSAT preparation and law school admissions advice from PowerScore Test Preparation.

User avatar
 Dave Killoran
PowerScore Staff
  • PowerScore Staff
  • Posts: 5853
  • Joined: Mar 25, 2011
|
#27095
Complete Question Explanation
(The complete setup for this game can be found here: lsat/viewtopic.php?f=325&t=6210)

The correct answer choice is (E)

The presence of K on each committee forces J to also to be a member of each committee per the third rule. This gives the planting committee a total of three members, closing it off to further members according to the condition in the question stem. From the fourth rule, then, M must be a member of the trails committee:
June 98_M12_game#1_L6_explanations_game#1_#4_diagram_1.png
As in question #3, the setup for the trails committee contains the minimum number of members; there could be other members of the trails committee. The planting committee is complete.

Answer choice (E) is proven correct by our analysis above. Note that answer choices (A) and (B) could never occur because each committee contains K, and F cannot be a member of the same committee as K per the second rule. Answer choices (C) and (D) could never occur because the question stem stipulates that the planting committee has exactly three members, and as discussed above those three members must be K, L, and J.
You do not have the required permissions to view the files attached to this post.

Get the most out of your LSAT Prep Plus subscription.

Analyze and track your performance with our Testing and Analytics Package.